Audit Final (8.1-13.4)

Pataasin ang iyong marka sa homework at exams ngayon gamit ang Quizwiz!

The primary source of information to be reported about litigation, claims, and assessments is the A. Client's management. B. Independent auditor. C. Court records. D. Client's legal counsel

Answer (A) is correct. According to AU-C 501, "Management is responsible for adopting policies and procedures to identify, evaluate, and account for litigation, claims, and assessments as a basis for the preparation of financial statements in accordance with the requirements of the applicable financial reporting framework." The auditor should discuss with management its policies and procedures for identifying and evaluating these matters.

Which of the following statements about evidence is true? A. A client's accounting records cannot be considered sufficient appropriate audit evidence on which to base the auditor's opinion. B. The cost of obtaining evidence is not an important consideration to an auditor in deciding what evidence should be obtained. C. Appropriate evidence supporting management's assertions should be conclusive rather than merely persuasive. D. Effective internal control contributes little to the reliability of the evidence created within the entity

Answer (A) is correct. Audit evidence consists of accounting records (initial entries and supporting records, such as ledgers, worksheets, and spreadsheets) and other information (minutes of meetings, confirmations, information obtained by inquiry, etc.). But accounting records alone do not provide sufficient appropriate evidence as a basis for an opinion on the financial statements.

Before performing substantive procedures at an interim date, an auditor should A. Consider the increased risk that period-end misstatements will not be detected. B. Assess control risk at a low level. C. Determine that materiality for the accounts tested is insignificant. D. Assess inherent risk at a high level

Answer (A) is correct. Certain factors should be considered before substantive procedures are performed at interim dates. For example, the auditor should consider the risk that period-end misstatements are not detected by performing appropriate substantive procedures or a combination of tests of controls and substantive procedures to cover the remaining period.

An auditor's purpose in reviewing the renewal of a note payable shortly after the balance sheet date most likely is to obtain evidence concerning relevant assertions about A. Classification and understandability. B. Valuation and allocation. C. Existence. D. Completeness

Answer (A) is correct. Events such as the renewal of the note payable do not require adjustment of the financial statements but may require disclosure. Accordingly, the auditor should determine that the renewal had essentially the same terms and conditions as the recorded debt at year end. A significant change may affect the classification of notes payable (e.g., as current or noncurrent), the understandability of the statements, and the required disclosures.

Which of the following types of audit evidence is the most persuasive? A. Bank statements obtained from the client. B. Client representation letter. C. Client worksheets supporting cost allocations. D. Prenumbered client purchase order forms

Answer (A) is correct. Evidence from independent sources outside the entity provides greater assurance of reliability for purposes of an audit than evidence secured or prepared solely within the entity. Although the bank statements are in the possession of the client, they originated outside of the client and, relative to the other responses, they are the most persuasive.

When confirmation for accounts payable is undertaken, which form of confirmation should be used? A. Positive confirmation. B. Negative confirmation. C. Either positive or negative confirmation, depending on the significance, or amount of the account. D. Neither positive nor negative confirmation; accounts payable cannot be confirmed

Answer (A) is correct. Ideally, the blank form of positive confirmation should be used. It requests that the balance due be provided by the creditor. External confirmation need not be performed if the auditor can become satisfied as to the existence of recorded payables using evidence available directly from the entity.

An auditor confirmed accounts receivable as of an interim date, and all confirmations were returned and appeared reasonable. Which of the following additional procedures most likely should be performed at year end? A. Review supporting documents for new large balances occurring after the interim date, and evaluate any significant changes in balances at year end. B. Resend confirmation requests for any significant customer balances remaining at year end. C. Send confirmation requests for all new customer balances incurred from the interim date to year end. D. Review cash collections subsequent to the interim date and the year end.

Answer (A) is correct. If incremental RMMs can be controlled, procedures to cover the remaining period ordinarily include (1) comparing information at the interim date with information at the balance sheet date to identify and investigate unusual amounts (e.g., new large balances) and (2) other analytical procedures or tests of details.

If the perpetual inventory records show lower quantities of inventory than the physical count, an explanation of the difference might be unrecorded A. Purchases. B. Purchase discounts. C. Sales discounts. D. Sales

Answer (A) is correct. In a perpetual system, purchases are debited directly to inventory at the time of the transaction rather than to a purchases account. A sale requires an immediate credit to inventory. Thus, failure to record a purchase would understate inventory.

If an auditor of an issuer examines purchase orders obtained from the issuer to verify proper authorization of transactions, then the auditor is conducting A. An inspection. B. A confirmation. C. An observation. D. A reperformance

Answer (A) is correct. Inspection is the examination of records or documents, whether internal or external, in paper, electronic, or other media. The auditor performs inspection of documents to verify proper authorization of transactions.

In auditing a manufacturing entity, which of the following procedures would an auditor least likely perform to determine whether slow-moving, defective, and obsolete items included in inventory are properly identified? A. Test the computation of standard overhead rates. B. Review inventory experience and trends. C. Compare inventory balances to anticipated sales volume. D. Tour the manufacturing plant or production facility

Answer (A) is correct. Testing the computation of standard overhead rates is relevant to the costing of inventory. It does not determine whether the underlying items are slow-moving, etc.

Which of the following procedures would be most appropriate for testing the completeness assertion as it applies to inventory? A. Performing cutoff procedures for shipping and receiving. B. Tracing inventory items from the tag listing back to the physical inventory quantities. C. Scanning perpetual inventory, production, and purchasing records. D. Examining paid vendor invoices

Answer (A) is correct. Testing the cutoff to consider the transfer of title of inventory in shipping and receiving is appropriate for testing the completeness assertion. The terms, FOB shipping point versus FOB destination, should be evaluated to assure that the goods were recorded in the proper period.

All of the following are examples of substantive tests to verify the valuation of net accounts receivable except the A. Inspection of accounts for current versus noncurrent status in the statement of financial position. B. Comparison of the allowance for bad debts with past records. C. Recomputation of the allowance for bad debts. D. Inspection of the aging schedule and credit records of past due accounts

Answer (A) is correct. The inspection of accounts for current versus noncurrent status is a test of management's assertion relating to statement presentation and disclosure. It is not a test of valuation.

During the process of confirming receivables as of December 31, Year 1, a positive confirmation was returned indicating the "balance owed as of December 31 was paid on January 9, Year 2." The auditor would most likely A. Determine whether a customary trade discount was taken by the customer. B. Determine whether any changes in the account occurred between January 1 and January 9, Year 2. C. Reconfirm the zero balance as of January 10, Year 2. D. Verify that the amount was received.

Answer (D) is correct. Responses to confirmation requests that involve significant differences are investigated by the auditor. Others are delegated to client employees with a request that explanations be given to the auditor. Such differences often arise because of recent cash payments. In that event, the auditor should trace remittances to verify that stated amounts were received.

Which of the following procedures would best detect a liability omission by management? A. Review and check mathematical accuracy of financial statements. B. Review articles of incorporation and corporate bylaws. C. Inquiry of senior support staff and recently departed employees. D. Review purchase contracts and other legal documents.

Answer (D) is correct. The auditor's search for unrecorded liabilities should include reading contracts, loan agreements, leases, correspondence from governmental agencies, and any legal documents in the client's possession.

Which of the following procedures would be most effective in reducing attestation risk? A. Discussion with responsible individuals. B. Inquiries of senior management. C. Analytical procedures. D. Examination of evidence.

Answer (D) is correct. To express an opinion, the practitioner must gather sufficient evidence to reduce attestation risk to an acceptably low level.

In testing plant and equipment balances, an auditor may inspect new additions listed on the analysis of plant and equipment. This procedure is designed to obtain evidence concerning relevant assertions about: Existence Classification and Understandability

existence Assertions about existence address whether assets or liabilities of the entity exist at a particular date. Assertions about classification and understandability concern whether financial statement components are appropriately presented, described, and disclosed (AU-C 315). Thus, inspection by the auditor provides direct evidence that new plant and equipment assets exist but is irrelevant to the classification and understandability assertions. Reading the financial statements and related notes provides evidence about these assertions.

When considering the use of management's written representations as audit evidence about the completeness assertion, an auditor should understand that such representations A. Complement, but do not replace, substantive procedures designed to support the assertion. B. Are not part of the evidence considered to support the assertion. C. Replace low assessed risks of material misstatement as evidence to support the assertion. D. Constitute sufficient appropriate evidence to support the assertion when considered in combination with sufficiently low assessed risks of material misstatement

Answer (A) is correct. AU-C 580 states that written representations provide necessary audit evidence that complements other audit procedures. However, they do not, by themselves, provide sufficient appropriate evidence about the matters to which they are relevant. Moreover, obtaining reliable written representations has no effect on the nature and extent of other procedures applied regarding (1) fulfillment of management's responsibilities or (2) specific assertions.

After gaining an understanding of a client's computer processing internal control, a financial statement auditor may decide not to test the effectiveness of the computer processing control procedures. Which of the following is not a valid reason for choosing to omit tests of controls? A. The assessment of the risks of material misstatement permits the auditor to rely on the controls. B. The time and dollar costs of testing exceed the time and dollar savings in substantive testing if the tests of controls show the controls to be effective. C. Risk assessment procedures have not identified relevant effective controls. D. The controls duplicate operative controls existing elsewhere in the system

Answer (A) is correct. Although controls appear to be effective based on the understanding of internal control, the auditor should perform tests of controls when the assessment of the RMMs at the relevant assertion level includes an expectation of their operating effectiveness. This expectation reflects the auditor's intention to rely on the controls in determining the nature, timing, and extent of substantive procedures.

In obtaining written representations from management, materiality limits ordinarily would apply to representations related to A. Amounts concerning related party transactions. B. Fraud involving members of management. C. The completeness of minutes of directors' meetings. D. The availability of financial records

Answer (A) is correct. An audit may not provide assurance that all related party transactions will be discovered. The auditor typically sets materiality limits to efficiently perform the audit.

Which of the following most likely would give the most assurance concerning the valuation assertion about accounts receivable? A. Assessing the allowance for uncollectible accounts for reasonableness. B. Inquiring about receivables pledged under loan agreements. C. Comparing receivable turnover ratios with industry statistics for reasonableness. D. Vouching amounts in the subsidiary ledger to details on shipping documents

Answer (A) is correct. Assertions about valuation concern whether balance sheet components have been included at appropriate amounts. One such assertion is that trade accounts receivable are stated at net realizable value (gross accounts receivable minus allowance for uncollectible accounts). Hence, assessing the allowance provides assurance about the valuation of the account.

After considering an entity's negative trends and financial difficulties, an auditor has substantial doubt about the entity's ability to continue as a going concern. The auditor's considerations relating to management's plans for dealing with the adverse effects of these conditions most likely would include management's plans to A. Increase ownership equity. B. Reduce existing lines of credit. C. Increase current dividend distributions. D. Purchase assets formerly leased

Answer (A) is correct. Once an auditor identifies conditions and events indicating that a substantial doubt exists about an entity's ability to continue as a going concern, (s)he should consider management's plans to mitigate their adverse effects. Increasing equity is likely to be a mitigating factor (AU-C 570). Thus, the auditor should consider the feasibility of such a plan, including arrangements to raise capital, and any arrangements to reduce dividends or to accelerate cash receipts from investors or affiliates.

Regardless of the assessed risks of material misstatement, an auditor should perform some A. Substantive procedures to restrict detection risk for significant transaction classes. B. Tests of controls to determine their effectiveness. C. Dual-purpose tests to evaluate both the risk of monetary misstatement and preliminary control risk. D. Analytical procedures to verify the design of controls

Answer (A) is correct. Regardless of the assessed RMMs (or the effectiveness of the relevant controls), the auditor should design and perform substantive procedures for all relevant assertions related to each material transaction class, account balance, and disclosure.

Which of the following procedures would yield the most reliable evidence? A. A recalculation of bad debt expense. B. A scanning of trial balances. C. An inquiry of client personnel. D. A comparison of beginning and ending retained earnings

Answer (A) is correct. Subject to exceptions, the presumption is that the independent auditor's direct personal knowledge, obtained through physical examination, observation, computation, and inspection, is more reliable than information obtained indirectly.

The objective of tests of details of transactions performed as substantive procedures is to A. Detect material misstatements at the relevant assertion level. B. Attain assurance about the reliability of the accounting system. C. Comply with generally accepted auditing standards. D. Evaluate whether management's policies and procedures operated effectively

Answer (A) is correct. Substantive procedures are (1) tests of the details of transaction classes, balances, and disclosures and (2) substantive analytical procedures. They are performed to detect material misstatements at the relevant assertion level. The auditor performs substantive procedures as a response to the related assessment of the RMMs (AU-C 330).

Of which of the following matters is a management representation letter required to contain specific representations? A. Information concerning fraud by the CFO. B. Reason for a significant increase in revenue over the prior year. C. Length of a material contract with a new customer. D. The competency and objectivity of the internal audit department

Answer (A) is correct. The CEO and the CFO usually sign the management representation letter. They state that they have no knowledge of any fraud or suspected fraud affecting the entity involving (1) management, (2) employees having significant roles in internal control, or (3) others if the fraud could materially affect the financial statements.

The accounts receivable turnover ratio increased significantly over a two-year period. This trend could indicate that A. The company is more aggressively collecting customer accounts. B. Customer sales have substantially decreased. C. The company has eliminated its discount policy. D. The accounts receivable aging has deteriorated

Answer (A) is correct. The accounts receivable turnover ratio equals net credit sales divided by average accounts receivable. Thus, the ratio increases if the entity more effectively collects accounts while holding other factors, such as credit policy, constant.

Which of the following audit procedures is least likely to detect an unrecorded liability? A. Analysis and recomputation of depreciation expense. B. Reading of the minutes of meetings of the board of directors. C. Mailing of standard bank confirmation forms. D. Analysis and recomputation of interest expense

Answer (A) is correct. The analysis and recomputation of depreciation expense is useful in determining whether the expense and asset accounts have been properly stated. Because liabilities are not part of the depreciation recording process, analysis and recomputation of depreciation would not detect unrecorded liabilities.

A nonissuer audit client failed to maintain copies of its procedures manuals and organizational flowcharts. What should the auditor do in an audit of financial statements? A. Adopt a substantive audit approach. B. Document the auditor's understanding of internal control. C. Express a qualified opinion on the basis of a scope limitation. D. Restrict the auditor's responsibility to assess the effectiveness of controls in the audit engagement letter.

Answer (A) is correct. The assessment of risks is a basis for choosing the audit approach. A substantive audit approach is based only on substantive procedures. A combined audit approach applies tests of controls and substantive procedures. For example, the risk assessment procedures may not identify effective controls for the relevant assertion, or testing controls may be inefficient, e.g., because client documentation is not available. The result is that the effect of controls is excluded from the risk assessment. In these cases, if the auditor adopts the substantive audit approach, (s)he needs to be satisfied that it will be effective in reducing audit risk to an acceptable level. For example, the substantive audit approach may not be feasible when the processing of routine transactions is highly automated with little manual intervention. In this case, the combined audit approach is chosen. Moreover, the auditor should design and perform some substantive procedures for all relevant assertions related to each material transaction class, account balance, or disclosure regardless of the assessment of the RMMs or the choice of audit approach.

In violation of a company policy, Lowell Company erroneously capitalized the cost of painting its warehouse. The auditor examining Lowell's financial statements would most likely detect this error when A. Examining the construction work orders supporting items capitalized during the year. B. Discussing capitalization policies with Lowell's controller. C. Observing, during the physical inventory observation, that the warehouse had been painted. D. Examining maintenance expense accounts

Answer (A) is correct. The audit plan for property, plant, and equipment includes verification of additions by vouching them to the original documents. The entries are traced from the journals back to authorizations, vendors' invoices, contracts, deeds, and construction work orders. Inspection of the work order for painting the warehouse should alert the auditor to the capitalization of an expense.

An auditor is least likely to test controls that provide for A. Classification of revenue and expense transactions by product line. B. Separation of the functions of recording disbursements and reconciling the bank account. C. Comparison of receiving reports and vendors' invoices with purchase orders. D. Approval of the purchase and sale of trading securities

Answer (A) is correct. The auditor is primarily concerned with the fairness of external financial reporting and therefore with controls relevant to a financial statement audit. (S)he is less likely to test controls over records used solely for internal management purposes than those used to prepare financial statements for external distribution. Assertions about the presentation of transactions by product line are not typically made. Thus, the auditor is unlikely to expend significant audit effort in testing such classifications.

The strongest criticism of the reliability of audit evidence that the auditor physically observes is that A. The auditor may not be qualified to evaluate the items (s)he is observing. B. Such evidence is too costly in relation to its reliability. C. The observation must occur at a specific time, which is often difficult to arrange. D. The client may conceal items from the auditor

Answer (A) is correct. The auditor may not be qualified to evaluate the quality or condition of the items observed, e.g., the quality of diamonds or the quantity of oil reserves. In such cases, the reliability of the evidence obtained is doubtful, and the auditor typically uses the work of an auditor's specialist. Moreover, observation does not verify the cost or ownership of assets. Ordinarily, observation is best suited to verifying the existence of an asset.

A portion of a client's inventory is in public warehouses. Evidence of the existence of this merchandise can most efficiently be acquired through which of the following methods? A. Confirmation. B. Observation. C. Inspection. D. Calculation

Answer (A) is correct. The auditor should confirm or investigate inventories held in public warehouses. Confirmation is efficient because of its low cost. The auditor ordinarily obtains confirmation of the existence of inventories by direct communication with the custodian. When a significant portion of current or total assets is held in a public warehouse, the auditor should consider testing the client's procedures by (1) evaluating the warehouser, (2) obtaining an independent accountant's report on the warehouser's internal control, (3) visiting the warehouse and observing physical counts, and (4) investigating the use of warehouse receipts (e.g., whether they are being used for collateral). The auditor should confirm with lenders the details of any pledged receipts.

Which of the following procedures should an auditor perform concerning litigation, claims, and assessments? A. Obtain a list from management that discloses all unasserted claims that it considers to be probable of assertion. B. Discuss with the client's legal counsel its philosophy of defending litigation, claims, and assessments that have a high probability of being resolved unfavorably. C. Inspect legal documents in the possession of the client's legal counsel that are relevant to pending litigation and unasserted claims and assessments. D. Confirm directly with the client's legal counsel that all litigation, claims, and assessments have been properly recorded in the financial statements

Answer (A) is correct. The auditor should obtain evidence about the possible loss from litigation, claims, and assessments. One procedure is to obtain a list from management describing and evaluating unasserted claims and assessments that (1) are probable of assertion and (2) have at least a reasonable possibility of an unfavorable outcome, with respect to which legal counsel has performed substantive legal services for the entity (AU-C 501).

An auditor may decide to perform only substantive procedures for certain assertions because the auditor believes A. Controls are not relevant to the assertions. B. The entity's control components are interrelated. C. Sufficient appropriate audit evidence to support the assertions is likely to be available. D. More emphasis on tests of controls than substantive tests is warranted

Answer (A) is correct. The auditor's risk assessment procedures may not have identified any suitably designed and implemented controls that are relevant to the assertions. Another possibility is that testing of controls may be inefficient. But the auditor needs to be satisfied that performing only substantive procedures will be effective in reducing audit risk to an acceptable level.

The element of the audit-planning process most likely to be agreed upon with the client before implementation of the audit strategy is the determination of the A. Timing of inventory observation procedures to be performed. B. Procedures to be undertaken to discover litigation, claims, and assessments. C. Evidence to be gathered to provide a sufficient basis for the auditor's opinion. D. Pending legal matters to be included in the inquiry of the client's attorney

Answer (A) is correct. The client is responsible for taking the physical inventory. The auditor is responsible for observing this process and performing test counts. The audit procedures are dependent upon management's plans. Thus, the auditor must coordinate the collection of this evidence with management.

In auditing contingent liabilities, which of the following procedures would an auditor most likely perform? A. Read the minutes of the board of directors' meetings. B. Perform tests of controls on the cash disbursement activities. C. Apply analytical procedures to accounts payable. D. Confirm the details of outstanding purchase order

Answer (A) is correct. The primary concern in an audit of liabilities is understatement. Under U.S. GAAP, a contingent liability depends on the occurrence of one or more future events. It must be recognized only if (1) it is probable that a liability has been incurred and (2) the amount can be reasonably estimated. Contingent liabilities commonly arise from (1) litigation, claims, and assessments; (2) guarantees; (3) warranties; (4) premiums and coupons; (5) environmental obligations; and (6) subsequent events. Among the procedures undertaken for other purposes that might disclose contingent liabilities is reading minutes of meetings of shareholders, directors, and committees held during and after the period audited.

An auditor performs a test to determine whether all merchandise for which the client was billed was received. The population for this test consists of all A. Vendors' invoices. B. Canceled checks. C. Merchandise received. D. Receiving reports

Answer (A) is correct. Vendors' invoices are the billing documents received by the client. They describe the items purchased, the amounts due, and the payment terms. The auditor should trace these invoices to the related receiving reports.

Which of the following procedures would an auditor most likely perform for year-end accounts receivable confirmations when the auditor did not receive replies to second requests? A. Inspect the shipping records documenting the merchandise sold to the debtors. B. Intensify the study of internal control concerning the revenue cycle. C. Increase the assessed level of detection risk for the existence assertion. D. Review the cash receipts journal for the month prior to year end

Answer (A) is correct. When customers fail to answer a second request for a positive confirmation, the accounts may be in dispute, uncollectible, or fictitious. The auditor should then apply alternative procedures (examination of subsequent cash receipts, shipping documents, and other client documentation of existence) to obtain evidence about the validity of nonresponding accounts.

Under which of the following circumstances would an entity be expected to accrue a loss contingency for the period under audit? A. The entity recorded the amount of an asset impaired as of the balance sheet date. B. The entity estimated the amount of a claim with a probable adverse outcome before issuance of the audit report. C. A reasonable estimate was determined for a liability incurred after the balance sheet date. D. Legal counsel communicated that an unfavorable judgment from current litigation was reasonably possible

Answer (B) is correct. A loss contingency is an existing condition, situation, or set of circumstances involving uncertainty as to possible loss that ultimately will be resolved when one or more future events occur or do not occur. A material contingent loss must be accrued (debit loss, credit liability or asset valuation allowance) when two conditions are met. Based on information available prior to the issuance (or availability for issuance) of the financial statements (and therefore the auditor's report issued with the statements), accrual is required if (1) it is probable that, at a balance sheet date, an asset has been impaired or a liability has been incurred and (2) the amount of the loss can be reasonably estimated.

A retail entity uses electronic data interchange (EDI) in executing and recording most of its purchase transactions. The entity's auditor recognizes that the documentation of the transactions will be retained for only a short period of time. To compensate for this limitation, the auditor most likely would A. Plan to make a 100% count of the entity's inventory at or near year end. B. Perform tests several times during the year, rather than only at year end. C. Increase the sample of EDI transactions to be selected for cutoff tests. D. Decrease the assessed risk of material misstatement for the existence or occurrence assertion

Answer (B) is correct. Accounting records and other evidence may be available only in electronic form. For example, if EDI is used, purchase, shipping, billing, cash receipt, and cash payment transactions often occur by exchange of electronic messages instead of source documents. In an image processing system, documents are scanned and converted into electronic form for storage purposes, and the source documents may not be retained. Thus, electronic evidence may exist at a given moment in time, but it may not be retrievable after a specified period if files are changed and no backups exist. Consequently, the auditor should consider the time during which information is available to determine the nature, timing, and extent of substantive tests and tests of controls. For example, the auditor may change the timing of audit tests by performing them several times during the year instead of only at year end.

During a recent audit of the revenue cycle, a CPA found the client had $1 million in accounts receivable recorded for fictitious customers. Which of the following tests most likely facilitated identification of the fraud? A. Reviewing the support for open sales orders not yet shipped at December 31. B. Sending positive confirmations to all of the client's customers with balances on December 31. C. Reviewing the segregation of duties for staff who had responsibility for sales, shipping, and invoicing. D. Examining the reconciliation between the subsidiary ledger and the general ledger control account

Answer (B) is correct. An external confirmation request is audit evidence obtained as a direct, written response from a third party (the confirming party). External confirmation of accounts receivable ordinarily is required. External confirmations are frequently relevant to assertions about account balances, especially (1) existence and (2) rights and obligations. Assertions about existence address whether assets, liabilities, or equity interests of the entity exist. Assertions about rights and obligations address whether (1) the entity holds or controls the rights to assets and (2) liabilities are the obligations of the entity. A positive confirmation request asks for a reply in all cases. It results in audit evidence only if a response is received. Thus, positive external confirmation requests provide relevant evidence that receivables exist (or do not exist) and that the client has the right of collection.

An auditor most likely would analyze inventory turnover rates to obtain evidence concerning relevant assertions about A. Rights and obligations. B. Valuation and allocation. C. Classification and understandability. D. Existence

Answer (B) is correct. Assertions about valuation and allocation address whether (1) assets, liabilities, and equity interests are included in the financial statements at appropriate amounts and (2) resulting adjustments are properly recorded. An examination of inventory turnover pertains to identifying slow-moving, excess, defective, and obsolete items included in inventories. This audit procedure tests the valuation and allocation assertion.

The most effective audit procedure for determining the collectibility of an account receivable is the A. Confirmation of the account. B. Review of the subsequent cash collections. C. Review of authorization of credit sales to the customer and the previous history of collections. D. Examination of the related sales invoice(s)

Answer (B) is correct. Collectibility pertains to the assertion of valuation. It is the principal issue with regard to the adequacy of the allowance for doubtful accounts. The best way to determine collectibility is to learn whether the receivable was subsequently collected. A confirmation provides evidence that a contract exists and that the debtor acknowledges the debt, but the subsequent collection of the receivable is the only means of gaining complete assurance that the amount will be paid.

A client maintains perpetual inventory records in both quantities and dollars. If the assessment of the risks of material misstatement is high, an auditor will probably A. Insist that the client perform physical counts of inventory items several times during the year. B. Request the client to schedule the physical inventory count at the end of the year. C. Apply gross profit tests to ascertain the reasonableness of the physical counts. D. Increase the extent of tests of controls relevant to the inventory cycle

Answer (B) is correct. If the assessment of the RMMs is high, the acceptable detection risk for a given level of audit risk decreases. The auditor should change the nature, timing, or extent of substantive procedures to increase the reliability and relevance of the evidence they provide. Thus, extending work done at an interim date to year end might be inappropriate. Observation of inventory at year end provides more reliable and relevant evidence.

Which of the following statements ordinarily is included among the written management representations obtained by the auditor in an audit of a nonissuer? A. Sufficient appropriate evidence has been made available to permit the expression of an unmodified opinion. B. Instances of fraud involving management that exceeded the materiality limits have been acknowledged. C. All transactions have been recorded in the accounting records. D. Management acknowledges that internal control has no material weaknesses

Answer (C) is correct. AU-C 580 lists the concerns ordinarily addressed in management representation letters, if applicable. The list includes disclosures about the recording of all transactions.

An auditor confirms a representative number of open accounts receivable as of December 31 and investigates respondents' exceptions and comments. By this procedure, the auditor would be most likely to learn of which of the following? A. One of the sales clerks has not been preparing charge slips for credit sales to family and friends. B. One of the cashiers has been covering a personal embezzlement by lapping. C. One of the computer control clerks has been removing all sales invoices applicable to his account from the data file. D. The credit manager has misappropriated remittances from customers whose accounts have been written off.

Answer (B) is correct. Lapping is the theft of a cash payment from one customer concealed by crediting that customer's account when a second customer makes a payment. When lapping exists at the balance sheet date, the confirmation of customer balances will probably detect the fraud because the customers' and entity's records of lapped accounts will differ.

To which of the following matters would materiality limits not apply when obtaining written client representations? A. Information about related party transactions. B. Instances of fraud involving management. C. Violations of state labor regulations. D. Disclosure of line-of-credit arrangements

Answer (B) is correct. Management's representations may be limited to matters that are considered individually or collectively material if management and the auditor have reached an understanding about the limits of materiality. Such limitations do not apply to certain representations not directly related to amounts in the financial statements, e.g., acknowledgment of responsibility for fair presentation, availability of records, and knowledge of fraud or suspected fraud affecting the entity involving (1) management, (2) employees with significant roles in internal control, or (3) others if the fraud could materially affect the statements (AU-C 580).

Which of the following events that occurred after a client's calendar-year end, but before the audit report date, would require disclosure in the notes to the financial statements, but no adjustment in the financial statements? A. Negotiations have resulted in compensation adjustments for union employees retroactive to the fourth quarter. B. New convertible bonds are issued to expand the company's product line. C. A fixed asset used in operations is sold at a substantial profit. D. A loss is reported on uncollectible accounts of an acknowledged distressed customer

Answer (B) is correct. New convertible bonds issued after year end do not require revision to the financial statements but do affect interpretation of the financial statements and disclosure in the notes.

Providing more supervision during an audit of a nonissuer in response to assessed risks of material misstatement at the financial statement level is an example of A. Further audit procedures. B. An overall response. C. Tests of controls. D. A substantive response

Answer (B) is correct. Overall responses apply to the assessed RMMs at the financial statement level. The following are examples of overall responses: (1) An emphasis on professional skepticism in evidence gathering and evaluation; (2) increased supervision; (3) assignment of staff with greater experience or expertise; (4) greater unpredictability in the choice of further audit procedures; and (5) changing the nature, timing, and extent of audit procedures, such as modifying the nature of a procedure to obtain more persuasive evidence (AU-C 330).

Which of the following elements ultimately determines the specific auditing procedures that are necessary in the circumstances to provide a reasonable basis for an opinion? A. Materiality. B. Auditor judgment. C. Audit risk. D. Reasonable assurance

Answer (B) is correct. Professional judgment involves applying relevant knowledge and experience to the facts and circumstances of an audit. The auditor exercises judgment to (1) interpret ethical requirements and GAAS and (2) make informed decisions, for example, about the sufficiency and appropriateness of audit evidence. The auditor also plans and performs the audit with professional skepticism. This is an attitude that includes (1) a questioning mind, (2) being alert to possible misstatements, and (3) a critical assessment of audit evidence (AU-C 500). Thus, judgment is required to select the specific auditing procedures needed to obtain that evidence

When performing a substantive test of a random sample of cash disbursements, an auditor is supplied with a photocopy of vendor invoices supporting the disbursements for one particular vendor rather than the original invoices. The auditor is told that the vendor's original invoices have been misplaced. What should the auditor do in response to this situation? A. Increase randomly the number of items in the substantive test to increase the reliance that may be placed on the overall test. B. Reevaluate the risk of fraud and design alternate tests for the related transactions. C. Increase testing by agreeing more of the payments to this particular vendor to the photocopies of its invoices. D. Count the missing original documents as misstatements, and project the total amount of the error based on the size of the population and the dollar amount of the errors

Answer (B) is correct. Several issues should cause the auditor to be suspicious. First, how could the client lose the original? Second, how did the client obtain a photocopy if the original was lost? Finally, and most importantly, photocopies are much less credible given the ease with which they can be altered. Thus, the auditor should reevaluate the risk of fraud.

Which of the following is least likely to indicate the need to increase the assurance provided by substantive testing? A. An increase in the assessed control risk. B. A decrease in the assessed inherent risk. C. A lower acceptable audit risk. D. A lower acceptable level of detection risk

Answer (B) is correct. Substantive procedures are performed to detect material misstatements in management's assertions. The nature, timing, and extent of substantive procedures are determined by the acceptable level of audit risk. For a given audit risk, the acceptable detection risk is inversely related to the assessed risks of material misstatement. The assessed RMMs are combined assessments of control risk and inherent risk. Thus, a decrease in the assessed inherent risk (1) decreases the assessed RMMs for a given assessed control risk, (2) increases the acceptable detection risk, and (3) does not indicate a need for more persuasive audit evidence (AU-C-200).

Which of the following statements best describes why an auditor would use only substantive procedures to evaluate specific relevant assertions and risks? A. The cost of substantive procedures will exceed the cost of testing the relevant controls. B. Testing the operating effectiveness of the relevant controls would not be efficient. C. The internal auditor already has tested the relevant controls and found them effective. D. The relevant internal control components are not well documented

Answer (B) is correct. The assessment of risks is a basis for choosing the audit approach. The risk assessment procedures may not identify effective controls for the relevant assertion, or testing controls may be inefficient. In these cases, the auditor may wish to use a substantive audit approach.

An auditor should obtain evidence relevant to all the following factors relevant to third-party litigation against a client except the A. Amount of potential loss. B. Jurisdiction in which the matter will be resolved. C. Probability of an unfavorable outcome. D. Period in which the underlying cause for legal action occurred

Answer (B) is correct. The auditor is least interested in determining where the particular matter in litigation will be resolved. The major issue is the effect of the litigation on the fair presentation of the financial statements. Accordingly, the auditor should obtain evidence relevant to (1) the period in which the underlying cause for legal action occurred, (2) the degree of probability of an unfavorable outcome, and (3) the amount or range of potential loss.

For which of the following matters should an auditor obtain written management representations? A. Management's acknowledgment of its responsibility for employees' violations of laws. B. Management's compliance with contractual agreements that may affect the financial statements. C. Management's knowledge of future plans that may affect the price of the entity's stock. D. Management's cost-benefit justifications for not correcting internal control weaknesses

Answer (B) is correct. The auditor should obtain written representations in the form of a management representation letter to complement other auditing procedures. In addition to certain required written representations, the auditor may decide that requesting certain other representations is necessary. These may include aspects of contracts that may affect the statements, including noncompliance (AU-C 580).

An auditor analyzes repairs and maintenance accounts primarily to obtain evidence in support of the relevant assertion that all A. Expenditures for property and equipment have been recorded in the proper period. B. Expenditures for property and equipment have not been charged to expense. C. Noncapitalizable expenditures for repairs and maintenance have been properly charged to expense. D. Noncapitalizable expenditures for repairs and maintenance have been recorded in the proper period

Answer (B) is correct. The auditor should vouch significant debits from the repairs and maintenance expense account to determine whether any should have been capitalized.

An entity's internal control requires that an approved voucher, a prenumbered purchase order, and a prenumbered receiving report accompany every check request. To determine whether checks are being issued for unauthorized expenditures, an auditor most likely would select items for testing from the population of all A. Receiving reports. B. Canceled checks. C. Approved vouchers. D. Purchase orders

Answer (B) is correct. The best procedure is to test whether any checks have been issued without vouchers, purchase orders, and receiving reports. An appropriate sample of canceled checks should be traced to the related supporting documentation. The checks should not have been written before the dates on the receiving reports.

Tests designed to detect purchases made before the end of the year that have been recorded in the subsequent year most likely would provide assurance about the relevant assertion regarding A. Existence. B. Cutoff. C. Classification and understandability. D. Valuation and allocation

Answer (B) is correct. The cutoff assertion is that transactions and events have been recorded in the proper period. To determine that all goods for which title has passed to the client at year end are recorded in inventory and accounts payable, a purchases cutoff test is appropriate.

Which of the following audit procedures, if used, should be combined with other audit procedures when testing the operating effectiveness of controls? A. Reperformance. B. Inquiry. C. Inspection. D. Observation

Answer (B) is correct. The following tests are presented in the order of the sufficiency and appropriateness of the evidence they ordinarily produce (least to most): (1) inquiry, (2) observation, (3) inspection of relevant documentation, and (4) reperformance of a control. Inquiry alone does not provide sufficient, appropriate evidence to support a conclusion about the effectiveness of a control.

Which of the following procedures would an auditor most likely perform in searching for unrecorded liabilities? A. Compare a sample of purchase orders issued just after year end with the year-end accounts payable trial balance. B. Vouch a sample of cash disbursements recorded just after year end to receiving reports and vendor invoices. C. Scan the cash disbursements entries recorded just before year end for indications of unusual transactions. D. Trace a sample of accounts payable entries recorded just before year end to the unmatched receiving report file

Answer (B) is correct. The greatest risk in the audit of payables is that unrecorded liabilities exist. Omission of an entry to record a payable is a fraud or error that is more difficult to detect than an inaccurate or false entry. The search for unrecorded payables should include (1) examining cash disbursements made after the balance sheet date and comparing them with the accounts payable trial balance, (2) sending confirmations to vendors with small and zero balances, and (3) reconciling payable balances with vendors' documentation.

Which of the following audit procedures is best for identifying unrecorded trade accounts payable? A. Reconciling vendors' statements to the file of receiving reports to identify items received just prior to the balance sheet date. B. Reviewing cash disbursements recorded subsequent to the balance sheet date to determine whether the related payables apply to the prior period. C. Investigating payables recorded just prior to and just subsequent to the balance sheet date to determine whether they are supported by receiving reports. D. Examining unusual relationships between monthly accounts payable balances and recorded cash payments

Answer (B) is correct. The greatest risk in the audit of payables is that unrecorded liabilities exist. Omission of an entry to record a payable is a misstatement that is more difficult to detect than an inaccurate or false entry. The search for unrecorded payables should (1) include examining cash payments made after the balance sheet date and comparing them with the accounts payable trial balance, (2) sending confirmations to vendors with small and zero balances, and (3) reconciling payable balances with vendors' documentation.

Legal counsel's response to an auditor's request for information regarding litigation, claims, and assessments will ordinarily contain which of the following? A. Confidential information that would be prejudicial to the entity's defense if publicized. B. A statement of concurrence with management's determination of which unasserted possible claims warrant specification. C. An explanation regarding limitations on the scope of the response. D. An assertion that the unasserted possible claims identified by the client represent all such claims of the entity.

Answer (C) is correct. AU-C 501 indicates that a statement regarding the nature and reasons for any limitation on legal counsel's response should be requested in the letter of inquiry. The letter is sent to legal counsel requesting confirmation of the information presented.

Which of the following statements about litigation, claims, and assessments extracted from a letter from a client's legal counsel is most likely to cause the auditor to request clarification? A. "I believe that the possible liability to the company is nominal in amount." B. "I believe that the action can be settled for less than the damages claimed." C. "I believe that the company will be able to defend this action successfully." D. "I believe that the plaintiff's case against the company is without merit."

Answer (B) is correct. The letter of inquiry requests, among other things, that legal counsel evaluate the likelihood of pending or threatened litigation, claims, and assessments. It also requests that legal counsel estimate, if possible, the amount or range of potential loss. Thus, the auditor is concerned about the amount of the expected settlement as well as the likelihood of the outcome. The statement that the action can be settled for less than the damages claimed is an example given in AU-C 501 of an evaluation that is unclear about the likelihood of an unfavorable outcome.

The safeguarding of inventory most likely includes A. Application of established overhead rates on the basis of direct labor hours or direct labor costs. B. Periodic reconciliation of detailed inventory records with the actual inventory on hand by taking a physical count. C. Analytical procedures for raw materials, goods in process, and finished goods that identify unusual transactions, theft, and obsolescence. D. Comparison of the information contained on the purchase requisitions, purchase orders, receiving reports, and vendors' invoices

Answer (B) is correct. The recorded accountability for inventory should be compared with existing inventory at reasonable intervals by taking a physical count. If inventory is susceptible to loss through fraud or error, the comparison should be made independently. An independent comparison is one made by persons not having responsibility for inventory custody or the authorization or recording of transactions.

An auditor most likely would make inquiries of production and sales personnel concerning possible obsolete or slow-moving inventory to support the relevant assertion about A. Rights and obligations. B. Valuation and allocation. C. Existence. D. Classification and understandability

Answer (B) is correct. The valuation and allocation assertion is directed towards whether inventory is recorded at lower of cost or market. The discovery of slow-moving, excess, defective, or obsolete inventory suggests that the cost of inventory be written down to market.

Which of the following procedures would an auditor most likely perform to identify unusual sales transactions? A. Examining duplicate sales invoices for credit approval by the credit manager. B. Performing a trend analysis of quarterly sales. C. Tracing credits in the accounts receivable ledger to source documentation. D. Tracing cash receipt entries to the bank statement deposit for amount and date

Answer (B) is correct. Trend analysis considers ratios or accounts over time. Quarterly sales analysis compared with last year's amounts or budgets will identify unusual sales transactions and raise questions that the auditor may want to address. Trend analysis is based on the assumption that performance will continue in line with previous performance or industry trends unless unusual transactions exist.

To gain assurance that all inventory items in a client's inventory listing schedule are valid, an auditor most likely would vouch A. Items listed in receiving reports and vendors' invoices to the inventory listing schedule. B. Items listed in the inventory listing schedule to inventory tags and the auditor's recorded count sheets. C. Inventory tags noted during the auditor's observation to items listed in receiving reports and vendors' invoices. D. Inventory tags noted during the auditor's observation to items listed in the inventory listing schedule.

Answer (B) is correct. Validity relates to the existence assertion. To determine that the items exist, the direction of testing should be from the schedule to the inventory tags and ultimately to the auditor's count sheet.

In verifying debits to perpetual inventory records of a nonmanufacturing firm, the auditor would be most interested in examining the purchase A. Orders. B. Invoices. C. Journal. D. Requisitions

Answer (B) is correct. Vendor invoices, which state the items purchased, the amount due, and the payment terms, document inventory cost when compared with purchase orders and receiving reports.

Under which of the following circumstances would it be advisable for the auditor to confirm accounts payable with creditors? A. Confirmation response is expected to be favorable and accounts payable balances are immaterial in amount. B. Creditor statements are not available and internal controls relating to accounts payable are unsatisfactory. C. The majority of accounts payable balances are with associated companies. D. Internal controls relating to accounts payable are effective and there is sufficient appropriate evidence on hand to minimize the risk of material misstatement

Answer (B) is correct. When the internal controls relevant to assertions about accounts payable are ineffective, the risk of material misstatement is increased. The greater the RMM, the greater the assurance required from substantive procedures related to an assertion. The auditor may need to change the nature, timing, or extent of substantive procedures and consider external confirmations. The auditor also should confirm accounts payable when (1) documentary evidence is lacking, (2) individual creditors have relatively large balances, (3) the client has made a major purchase from the creditor regardless of the size of the balance, (4) unusual transactions are involved, or (5) the account is secured.

Evidence supporting the financial statements consists of the accounting records and all other information available to the auditor. Which of the following is an example of other information? A. General and subsidiary ledgers. B. Reconciliations. C. Minutes of meetings. D. Worksheets supporting cost allocations

Answer (C) is correct.

The primary reason an auditor requests letters of inquiry be sent to a client's legal counsel is to provide the auditor with A. Legal counsel's opinion of the client's historical experiences in recent similar litigation. B. The probable outcome of asserted claims and pending or threatened litigation. C. Corroboration of the information furnished by management about litigation, claims, and assessments. D. A description and evaluation of litigation, claims, and assessments that existed at the balance sheet date.

Answer (C) is correct. A letter of inquiry to a client's external legal counsel is the auditor's primary means of corroborating information furnished by management about litigation, claims, and assessments. If in-house legal counsel is primarily responsible for the entity's litigation, claims, and assessments, the auditor should send a similar letter of inquiry to in-house legal counsel. But the letter to in-house legal counsel is not a substitute for direct communication with external legal counsel.

If an auditor is obtaining an understanding of an issuer's information and communication component of internal control, which of the following factors should the auditor assess? A. The integrity and ethical values of top management. B. The oversight responsibility over financial reporting and internal control by the board or audit committee. C. The classes of transactions in the issuer's operations that are significant to the issuer's financial statements. D. The philosophy and operating style of management to promote effective internal control over financial reporting

Answer (C) is correct. According to AS 2110, the auditor should obtain a sufficient understanding of each component of internal control over financial reporting to (1) identify the types of misstatements, (2) assess the factors that affect the risks of material misstatement, and (3) design further audit procedures. Thus, the auditor should obtain an understanding of the information system, including the related business processes, relevant to financial reporting. This understanding includes the classes of transactions in the issuer's operations that are significant to the issuer's financial statements.

Which of the following presumptions is least likely to relate to the reliability of audit evidence? A. Evidence obtained from independent sources outside the entity is more reliable than evidence secured solely within the entity. B. The more effective internal control is, the more assurance it provides about the accounting data and financial statements. C. An auditor's opinion is formed within a reasonable time to achieve a balance between benefit and cost. D. The auditor's direct personal knowledge obtained through observation and inspection is more persuasive than information obtained indirectly

Answer (C) is correct. Appropriate audit evidence is relevant and reliable. Evidence is usually more reliable when it (1) is obtained from independent sources; (2) is generated internally under effective internal control; (3) is obtained directly by the auditor; (4) is in documentary form, whether paper, electronic, or other medium; and (5) consists of original documents. However, the need for (1) reporting to be timely and (2) maintaining a balance between benefit and cost are inherent limitations of the audit. Thus, for the opinion to be relevant, it must be formed within a reasonable period of time.

An auditor most likely would limit substantive audit tests of sales transactions when the risks of material misstatement are assessed as low for the existence and occurrence assertions concerning sales transactions and the auditor has already gathered evidence supporting A. Opening and closing inventory balances. B. Shipping and receiving activities. C. Cash receipts and accounts receivable. D. Cutoffs of sales and purchases

Answer (C) is correct. Cash receipts and accounts receivable have a direct relationship with sales. A cash sale results in a debit to cash and a credit to sales. A sale on account results in a debit to accounts receivable and a credit to sales. Thus, evidence related to cash receipts and accounts receivable provides assurances about sales.

A weakness in internal control over recording retirements of equipment may cause an auditor to A. Review the subsidiary ledger to ascertain whether depreciation was taken on each item of equipment during the year. B. Inspect certain items of equipment in the plant and trace those items to the accounting records. C. Select certain items of equipment from the accounting records and locate them in the plant. D. Trace additions to the "other assets" account to search for equipment that is still on hand but no longer being used

Answer (C) is correct. Failure to record retirements results in overstating equipment in the subsidiary records because the physical assets are not in the entity's possession. Thus, vouching items from the accounting records by locating the physical assets they represent provides evidence of whether retirements are unrecorded.

Tests of controls in a financial statement audit are least likely to be omitted with regard to A. Accounts believed to be subject to ineffective controls. B. Subsequent events. C. Accounts representing many transactions. D. Accounts representing few transactions

Answer (C) is correct. For high-volume accounts, the auditor ordinarily tests controls because of efficiency considerations. The auditor tests suitably designed controls given an expectation that controls are operating with some degree of effectiveness.

In performing a search for unrecorded retirements of fixed assets, an auditor most likely would A. Tour the client's facilities, and then analyze the repair and maintenance account. B. Analyze the repair and maintenance account, and then tour the client's facilities. C. Inspect the property ledger and the insurance and tax records, and then tour the client's facilities. D. Tour the client's facilities, and then inspect the property ledger and the insurance and tax records

Answer (C) is correct. In a search for unrecorded retirements, that is, a test of the completeness assertion, the auditor should first determine from the property ledger what assets are recorded and then tour the facilities to determine whether those assets are physically present. The completeness assertion addresses whether all transactions that should be presented (e.g., retirements) are included in the statements. However, in this case, the completeness assertion is closely related to the existence assertion.

To test the effectiveness of controls, an auditor ordinarily selects from a variety of techniques, including A. Inquiry and analytical procedures. B. Inspection and verification. C. Reperformance and observation. D. Comparison and confirmation

Answer (C) is correct. Inquiry alone is not sufficient to test the operating effectiveness of controls. Other audit procedures performed in combination with inquiry may include inspection, recalculation, and reperformance of a control that pertains to an assertion.

To measure how effectively an entity employs its resources, an auditor calculates inventory turnover by dividing average inventory into A. Net sales. B. Operating income. C. Cost of goods sold. D. Gross sales

Answer (C) is correct. Inventory turnover equals cost of goods sold divided by average inventory. It provides a measure of how many times inventory requires replacement.

An auditor is determining whether internal control relative to the revenue cycle of a wholesaling entity is operating effectively in minimizing the failure to prepare sales invoices. The auditor most likely would select a sample of transactions from the population represented by the A. Customer order file. B. Cash receipts file. C. Shipping document file. D. Sales invoice file

Answer (C) is correct. Matching shipping documents to sales invoices will indicate whether the shipping documents generated the preparation of a sales invoice.

Cooper, CPA, believes there is substantial doubt about the ability of Zero Corp. to continue as a going concern for a reasonable period of time. In evaluating Zero's plans for dealing with the adverse effects of future conditions and events, Cooper most likely will consider, as a mitigating factor, Zero's plans to A. Strengthen internal controls over cash disbursements. B. Purchase production facilities currently being leased from a related party. C. Postpone expenditures for research and development projects. D. Discuss with lenders the terms of all debt and loan agreements

Answer (C) is correct. Once an auditor has identified conditions and events indicating that substantial doubt exists about an entity's ability to continue as a going concern, the auditor should first obtain written representations and then consider management's plans to mitigate their adverse effects. The auditor should consider plans to dispose of assets, borrow money or restructure debt, reduce or delay expenditures, and increase equity.

The adverse effects of events causing an auditor to believe there is substantial doubt about an entity's ability to continue as a going concern would most likely be mitigated by evidence relating to the A. Committed arrangements to convert preferred stock to noncurrent debt. B. Ability to expand operations into new product lines in the future. C. Marketability of assets that management plans to sell. D. Feasibility of plans to purchase leased equipment at less than market value

Answer (C) is correct. Once an auditor identifies conditions and events indicating that a substantial doubt exists about an entity's ability to continue as a going concern, (s)he should consider management's plans to mitigate their adverse effects. With regard to plans to sell assets, the auditor's considerations may include restrictions on disposal, marketability of the assets, and the possible direct and indirect effects of the disposal. Thus, the greater the marketability of the assets, the greater the potential mitigation.

An internal control narrative indicates that an approved voucher is required to support every check request for payment of merchandise. Which of the following procedures provides the greatest assurance that this control is operating effectively? A. Select and examine vouchers and ascertain that the related canceled checks are dated no earlier than the vouchers. B. Select and examine vouchers and ascertain that the related canceled checks are dated no later than the vouchers. C. Select and examine canceled checks and ascertain that the related vouchers are dated no later than the checks. D. Select and examine canceled checks and ascertain that the related vouchers are dated no earlier than the checks.

Answer (C) is correct. Payment vouchers bearing the required approvals should be supported by a properly authorized purchase requisition, a purchase order executing the transaction, a receiving report indicating all goods ordered have been received in good condition, and a vendor invoice confirming the amount owed. To determine that check requests are valid, the appropriate audit procedure is therefore to compare checks and the related vouchers. The direction of testing should be from a sample of checks to the approved vouchers. If the date of a voucher is later than the date of the related check, the inference is that a check was issued without proper support.

In connection with the annual audit, which of the following is not a subsequent events procedure? A. Review the latest subsequent interim financial statements. B. Discuss with officers the current status of items in the financial statements that were accounted for on the basis of preliminary or inconclusive data. C. Make inquiries with respect to the financial statements covered by the auditor's previously issued report if new information has become available during the current audit that might affect that report. D. Read available minutes of meetings of shareholders, directors, and committees. With regard to meetings for which minutes are not available, inquire about matters dealt with at such meetings

Answer (C) is correct. Subsequent events are events or transactions occurring between the balance sheet date and the date of the auditor's report (AU-C 560). Making inquiries with respect to financial statements covered by the auditor's previously issued report when new information becomes available during the current audit is not a subsequent events procedure. Such inquiries pertain to previously undiscovered facts in existence at the date of the prior report that affect the report.

After obtaining an understanding of internal control for the audit of a nonissuer's financial statements, an auditor decided not to perform tests of controls. The auditor most likely decided that A. The assessed inherent risk exceeded the assessed control risk. B. A reduction in the assessed risks of material misstatement is justified for certain financial statement assertions. C. Performing only substantive procedures would effectively reduce audit risk to an acceptably low level. D. The available evidence obtained through tests of controls would not support higher assessed risks of material misstatement

Answer (C) is correct. The assessment of risks is a basis for choosing the audit approach. A substantive audit approach is based only on substantive procedures.

Which of the following statements about audit evidence is true? A. To be appropriate, audit evidence should be either persuasive or relevant but need not be both. B. The difficulty and expense of obtaining audit evidence about an account balance is a valid basis for omitting the test. C. The sufficiency and appropriateness of audit evidence is a matter of professional judgment. D. A client's accounting records can be sufficient audit evidence to support the financial statements

Answer (C) is correct. The auditor exercises professional judgment when forming a conclusion about whether sufficient appropriate audit evidence has been obtained to reduce audit risk to an acceptably low level. Sufficiency measures the quantity of audit evidence. Appropriateness measures its quality (relevance and reliability). To form this conclusion, the auditor considers all relevant evidence, regardless of whether it corroborates or contradicts the assertions in the statements.

The auditor who interviews the plant manager is most likely to rely upon this interview as primary support for an audit conclusion on A. Capitalization vs. expensing policy. B. Allocation of fixed and variable costs. C. The necessity to record a provision for deferred maintenance costs. D. The adequacy of the depreciation expense

Answer (C) is correct. The auditor typically does not use the responses to inquiries as primary support for an audit conclusion. However, the determination by management that a liability exists should convince the auditor that an entry should be made.

Which of the following management assertions is an auditor most likely testing if the audit objective states that all inventory on hand is reflected in the ending inventory balance? A. Inventory is properly presented in the financial statements. B. Inventory is properly valued. C. Inventory is complete. D. The entity has rights to the inventory

Answer (C) is correct. The completeness assertion about account balances at the end of a period is that all assets, liabilities, and equity interests that should have been recorded have been recorded. The completeness assertion for inventory is to determine whether the balance contains inventory (1) on hand and (2) owned by the entity that is in transit or stored at outside locations.

Cooper, CPA, is auditing the financial statements of a small rural municipality. The receivable balances represent residents' delinquent real estate taxes. Internal control at the municipality is ineffective. To determine the existence of the accounts receivable balances at the balance sheet date, Cooper would most likely A. Send negative confirmation requests. B. Inspect the internal records such as copies of the tax invoices that were mailed to the residents. C. Send positive confirmation requests. D. Examine evidence of subsequent cash receipts.

Answer (C) is correct. The presumption that the auditor will request confirmation of receivables cannot be overcome unless (1) the receivables are not material, (2) external confirmation requests are unlikely to be effective, (3) the assessed risks of material misstatement are low, and (4) other planned substantive procedures apply to the assessed risks. None of these conditions apply. Thus, the auditor should confirm the receivables. However, negative confirmation requests are not used unless the assessed risks of material misstatement are low. Because the municipality's internal control is ineffective, the auditor should send positive confirmation requests.

Which of the following procedures most likely would assist an auditor to identify litigation, claims, and assessments? A. Obtain a letter of representations from the client's underwriter of securities. B. Inspect checks included with the client's cutoff bank statement. C. Read the file of correspondence from taxing authorities. D. Apply ratio analysis on the current-year's liability accounts

Answer (C) is correct. To obtain evidence relevant to circumstances indicating litigation, claims, and assessments, the auditor should inquire of management. The auditor also should communicate directly with legal counsel. Other relevant procedures include risk assessment procedures. Examples include (1) reading minutes of meetings of shareholders, directors, and appropriate committees; (2) reading contracts, loan agreements, leases, and correspondence from governmental agencies; (3) obtaining information about guarantees from bank confirmations; and (4) inspecting other documents for possible guarantees by the client. Thus, the communication with the taxing authorities may reveal possible tax assessments excluded by the client.

The scope of an audit is not restricted when legal counsel's response to an auditor as a result of a client's letter of inquiry limits the response to A. The probable outcome of asserted claims and pending or threatened litigation. B. The legal counsel's opinion of the entity's historical experience in recent similar litigation. C. Matters to which the legal counsel has given substantive attention in the form of legal representation. D. An evaluation of the likelihood of an unfavorable outcome of the matters disclosed by the entity

Answer (C) is correct. Two limitations on an entity's external legal counsel's response are not scope limitations. The response may be limited to matters to which the legal counsel has given substantive attention in the form of legal consultation or representation. Also, the response may be limited to matters that are individually or collectively material, such as when the entity and the auditor have agreed on materiality limits, and management has stated the limits in the letter of inquiry.

An auditor determines that a client has properly capitalized a leased asset (and corresponding lease liability) as representing, in substance, an installment purchase. As part of the auditor's procedures, (s)he should A. Determine that the leased property is being amortized over the life of the lease. B. Evaluate whether the total amount of lease payments represents the fair value of the property. C. Evaluate the propriety of the interest rate used in discounting the future lease payments. D. Substantiate the cost of the property to the lessor and determine that this is the cost recorded by the client.

Answer (C) is correct. Under U.S. GAAP, a capital lease is recorded by the lessee as an asset and a liability at the present value of the minimum lease payments. Thus, the interest rate used in the discounting process is an important consideration in determining whether the asset is fairly presented in the balance sheet. The interest rate is the lessee's incremental borrowing rate, unless the lessor's implicit rate in the lease is known and is less than the lessee's incremental rate.

When auditing a public warehouse, which of the following is the most important audit procedure with respect to disclosing unrecorded liabilities? A. Review of outstanding receipts. B. Observation of inventory. C. Inspection of receiving and issuing procedures. D. Confirmation of negotiable receipts with holders

Answer (C) is correct. When auditing a public warehouse, the inspection of receiving and issuing procedures is the most important procedure for disclosing unrecorded liabilities. Shipping orders and receiving reports that are not reflected in the records suggest that transactions are not being properly recorded.

When compared with a nineteenth-century auditor, today's auditor places less relative emphasis upon A. Physical observation. B. Overall tests of ratios and trends. C. External confirmation. D. Examination of documentary support.

Answer (D) is correct.

If the auditor determines that an inquiry of a client's external legal counsel is necessary, who should make the inquiry? A. The auditor's attorney. B. The auditor. C. The stockholders. D. Client management.

Answer (D) is correct. A letter of inquiry to a client's legal counsel is the auditor's primary means of corroborating information furnished by management about litigation, claims, and assessments. Evidence obtained from the client's legal department may provide the needed corroboration, but it does not substitute for information that external legal counsel may refuse to furnish. Thus, the auditor should request management to send a letter of inquiry to legal counsel with whom management consulted. Without the client's consent, legal counsel may not respond (AU-C 501).

Which of the following procedures is least likely to be performed before the balance sheet date? A. Observation of inventory. B. Testing internal control over cash. C. Confirmation of receivables. D. Search for unrecorded liabilities.

Answer (D) is correct. A significant risk is that all payables may not be reflected in the year-end balance. The auditor will review cash disbursements made subsequent to year end to determine whether payments are for previously unrecorded liabilities. Other procedures that would not be performed prior to the balance sheet date include reviewing subsequent events, requesting the lawyer's letter, and obtaining management representations.

If the auditor intends to rely on the operating effectiveness of relevant controls, which test of controls is necessary to obtain sufficient appropriate evidence? A. Reperformance. B. Inspection. C. Observation. D. Inquiry.

Answer (D) is correct. According to AU-C 330, an auditor's tests of controls include other audit procedures in combination with inquiry. For this purpose, inquiry combined with inspection, reperformance, or recalculation may be preferable to inquiry and observation. Observation is relevant only at a moment in time.

A client uses a suspense account for unresolved questions whose final accounting has not been determined. If a balance remains in the suspense account at year-end, the auditor would be most concerned about A. Suspense credits that the auditor determines to be customer deposits. B. Suspense debits that the auditor verifies will have realizable value to the client. C. Suspense credits that management believes should be classified as "current liability." D. Suspense debits that management believes will benefit future operations.

Answer (D) is correct. Although the auditor must evaluate relevant assertions about all accounts, the greatest risks are overstated assets and understated liabilities. The unverified suspense debits represent assets that may not exist.

An auditor's purpose in reviewing credit ratings of customers with delinquent accounts receivable most likely is to obtain evidence concerning relevant assertions about A. Rights and obligations. B. Classification and understandability. C. Existence. D. Valuation and allocation.

Answer (D) is correct. Assertions about valuation and allocation address whether (1) assets, liabilities, and equity interests are included in the financial statements at appropriate amounts and (2) resulting adjustments are properly recorded. Determining the net realizable value of accounts receivable includes assessing whether the client's allowance for uncollectibility is reasonable. Reviewing the credit ratings of delinquent customers provides evidence for that purpose.

An auditor reconciles the total of the accounts receivable subsidiary ledger to the general ledger control account, as of October 31. By this procedure, the auditor would be most likely to learn of which of the following? A. An October check from a customer was posted in error to the account of another customer with a similar name. B. An account balance is past due and should be written off. C. An October invoice was improperly computed. D. An opening balance in a subsidiary ledger account was improperly carried forward from the previous accounting period.

Answer (D) is correct. By reconciling the accounts receivable ledger to the general ledger control account, transfer misstatements will be identified.

Which of the following procedures would an auditor most likely perform to obtain assurance that slow-moving and obsolete items included in inventories are properly identified? A. Tracing inventory observation test counts to perpetual listings. B. Confirming inventories at locations outside the entity's premises. C. Testing shipping and receiving cutoff procedures. D. Examining an analysis of inventory turnover.

Answer (D) is correct. Calculating inventory turnover is an analytical procedure. The ratio equals the cost of sales divided by the average inventory. A high turnover implies that the entity does not hold stocks of inventories that are unproductive (slow moving, excess, defective, or obsolete). This procedure tests the valuation assertion.

The auditing standards define external confirmation as "a direct written response to the auditor from a third party (the confirming party), either in paper form or by electronic or other medium." The assertions for which confirmation of accounts receivable balances provides primary evidence are A. Valuation and rights and obligations. B. Completeness and valuation. C. Existence and completeness. D. Rights and obligations and existence.

Answer (D) is correct. Confirmation by means of direct (independent) communication with debtors is the generally accepted auditing procedure for accounts receivable. Properly designed requests may address any assertion in the financial statements, but they are most likely to be effective for the existence and rights and obligations assertions. Thus, confirmation provides evidence that (1) receivables are valid, (2) the client has ownership of the accounts and the right of collection, and (3) the debtor has the obligation to pay.

The audit of year-end physical inventories should include steps to verify that the client's purchases and sales cutoffs were adequate. The audit steps should be designed to detect whether merchandise included in the physical count at year end was not recorded as a A. Purchase in the current period. B. Sale in the subsequent period. C. Purchase return in the subsequent period. D. Sale in the current period.

Answer (D) is correct. Goods on hand and counted in the year-end inventory also should not have been recorded as sold during the current audit period. If they were sold, they could not have been owned by the client at year end. The auditor should perform cutoff tests to assure the proper recording of year-end inventory. These tests include comparison of the records of sales and purchases for several days before and after the balance sheet date with duplicate sales invoices and shipping records. Items purchased and items not yet sold should be in inventory. Items sold or not yet purchased should not be in inventory.

If the objective of an auditor's test of details is to detect a possible understatement of sales, the auditor most likely would trace transactions from the A. Sales journal to the cash receipts journal. B. Sales invoices to the shipping documents. C. Cash receipts journal to the sales journal. D. Shipping documents to the sales invoices.

Answer (D) is correct. If a shipment occurred, matching shipping documents to recorded sales would disclose the understatement if no recorded sale could be found.

Which of the following explanations might satisfy an auditor who discovers significant debits to an accumulated depreciation account? A. An asset has been recorded at its fair value. B. Prior years' depreciation charges were erroneously understated. C. An appropriation for possible loss on retirement has been recorded. D. Extraordinary repairs have lengthened the life of an asset.

Answer (D) is correct. If extraordinary (major) repairs have lengthened the life of an asset without improving its quality or quantity, the entity may reduce the amount of accumulated depreciation to increase the net carrying amount of the asset. This practice is acceptable for representing the increased life of an asset. It also is often used to account for replacements.

At the conclusion of an audit, an auditor is reviewing the audit evidence obtained. With regard to the valuation of inventory, the auditor concludes that the evidence obtained is not sufficient to support management's assertions. Which of the following actions is the auditor most likely to take? A. Obtain a statement from management supporting the inventory valuation. B. Consult with the audit committee and issue a disclaimer of opinion. C. Consult with the audit committee and issue a qualified opinion. D. Obtain additional evidence regarding the valuation of inventory.

Answer (D) is correct. If the audit evidence is not sufficient and appropriate, the auditor should perform other procedures, such as tests of details or analytical procedures.

Which of the following is true related to the auditor's consideration of controls? A. The auditor should consider testing and documenting the efficiency of the entity's controls related to relevant assertions. B. A material misstatement detected by the auditor, but not detected by the entity, should be considered a material weakness in internal control. C. The absence of misstatements detected by an auditor's substantive procedures should be considered evidence that controls related to the relevant assertion being tested are effective. D. Misstatements detected by the auditor's substantive procedures should be considered when testing the effectiveness of related controls.

Answer (D) is correct. Misstatements detected by the auditor's substantive procedures should be considered when testing the effectiveness of related controls. The auditor may need to assess the RMMs at a higher level if fraud or error is detected that should have been detected by the entity's controls.

Which of the following might be detected by an auditor's review of the client's sales cutoff? A. Lapping of year-end accounts receivable. B. Unrecorded sales discounts. C. Excessive goods returned for credit. D. Inflated sales for the year.

Answer (D) is correct. Sales cutoff tests are designed to detect the client's manipulation of sales. By examining recorded sales for several days before and after the balance sheet date and comparing them with sales invoices and shipping documents, the auditor may detect the recording of a sale in a period other than that in which title passed.

Which of the following procedures should an auditor ordinarily perform regarding subsequent events? A. Send second requests to the client's customers who failed to respond to initial accounts receivable confirmation requests. B. Communicate material weaknesses in internal control to the client's audit committee. C. Review the cutoff bank statements for several months after the year end. D. Read the latest subsequent interim financial statements.

Answer (D) is correct. Subsequent events procedures include (1) reading the latest subsequent interim statements, if any; (2) inquiring of management and those charged with governance about the occurrence of subsequent events and various financial and accounting matters; (3) reading the minutes of meetings of owners, management, and those charged with governance; (4) obtaining a letter of representations from management; (5) inquiring of client's legal counsel; and (6) obtaining an understanding of management's procedures for identifying subsequent events

Which of the following procedures will an auditor most likely perform to obtain evidence about the occurrence of subsequent events? A. Confirming bank accounts established after year end. B. Recomputing a sample of large-dollar transactions occurring after year end for arithmetic accuracy. C. Investigating changes in equity occurring after year end. D. Inquiring of the entity's legal counsel concerning litigation, claims, and assessments arising after year end.

Answer (D) is correct. Subsequent events procedures include (1) reading the latest subsequent interim statements, if any; (2) inquiring of management and those charged with governance about the occurrence of subsequent events and various financial and accounting matters; (3) reading the minutes of meetings of owners, management, and those charged with governance; (4) obtaining a letter of representations from management; (5) inquiring of client's legal counsel; and (6) obtaining an understanding of management's procedures for identifying subsequent events.

The primary difference between an audit of the balance sheet and an audit of the income statement is that the audit of the income statement deals with the verification of A. Authorizations. B. Costs. C. Cutoffs. D. Transactions.

Answer (D) is correct. The audit of the income statement focuses on the propriety of handling transactions because most income statement accounts represent large volumes of transactions. The audit of the balance sheet concentrates on verification of account balances.

A client's procurement system ends with the assumption of a liability and the eventual payment of the liability. Which of the following best describes the auditor's primary concern with respect to liabilities resulting from the procurement system? A. Acquisition of materials is not made from one vendor or one group of vendors. B. Authority to incur liabilities is restricted to one designated person. C. Commitments for all purchases are made only after established competitive bidding procedures are followed. D. Accounts payable are not materially understated.

Answer (D) is correct. The auditor is primarily concerned that accounts payable are not materially understated on the balance sheet, i.e., that accounts payable are recorded at the proper amounts. This is important because current liabilities are significant in evaluating an entity's short-term solvency. Also, understatement of accounts payable understates expenses. The auditor should guard against management's tendency both to inflate earnings and to exaggerate the financial strength of the firm through understatement of liabilities and expenses or overstatement of assets and revenues.

After obtaining an understanding of internal control and assessing the risks of material misstatement in a financial statement audit, an auditor decided to perform tests of controls. The auditor most likely decided that A. An increase in the assessed level of control risk is justified for certain financial statement assertions. B. Additional evidence to support an expectation of operating effectiveness is not available. C. There were many internal control weaknesses. D. It would be efficient to perform tests of controls that would result in a reduction in substantive procedures.

Answer (D) is correct. The auditor may rely on the operating effectiveness of controls in determining the nature, timing, and extent of substantive procedures. Thus, the auditor should weigh the cost of performing tests of controls against the expected savings from devoting less audit effort to substantive testing

Soon after Boyd's audit report was released, Boyd learned of certain related party transactions that occurred during the year under audit. These transactions were not disclosed in the notes to the financial statements. Boyd should A. Ask the client to disclose the transactions in subsequent interim statements. B. Plan to audit the transactions during the next engagement. C. Recall all copies of the audited financial statements. D. Discuss the matter with management.

Answer (D) is correct. The auditor should (1) discuss the matter with management and those charged with governance and (2) determine whether the financial statements need revision and, if so, inquire how management intends to address the matter in the statements (AU-C 560).

Which of the following most likely should be included as part of an auditor's tests of controls? A. Reconciliation. B. Analytical procedures. C. External confirmations. D. Inspection.

Answer (D) is correct. The auditor should perform other procedures in combination with inquiry to obtain evidence about the operating effectiveness of controls. Thus, inquiry by itself is not sufficient. Accordingly, inquiry combined with inspection, recalculation, or reperformance may be preferable to inquiry and observation. An observation is relevant only at a moment in time (AU-C 330). Inspection is an examination of internal or external records or documents in any medium. Inspection also includes physical examination of an asset (AU-C 500).

An auditor suspects that certain client employees are ordering merchandise for themselves over the Internet without recording the purchase or receipt of the merchandise. When vendors' invoices arrive, one of the employees approves the invoices for payment. After the invoices are paid, the employee destroys the invoices and the related vouchers. In gathering evidence regarding the fraud, the auditor most likely would select items for testing from the file of all A. Vendors' invoices. B. Receiving reports. C. Approved vouchers. D. Cash disbursements.

Answer (D) is correct. The best procedure to test whether any checks have been issued without supporting vouchers, purchase orders, and receiving reports is to select an appropriate sample of canceled checks (cash disbursements) and trace them to the related supporting documentation

Which of the following procedures would be appropriate to test the existence assertion during an audit of accounts receivable? A. Trace a sample of invoices to recording in the general ledger. B. Trace transactions from the subsidiary ledger to the general ledger. C. Determine that all shipments before year end are recorded as sales. D. Send confirmations to customers.

Answer (D) is correct. The existence assertion about account balances at period end is that assets (e.g., accounts receivable), liabilities, and equity interests exist. Thus, external confirmation of accounts receivable by sending requests to customers tests for existence. They must be confirmed unless (1) they are immaterial, (2) confirmation would be ineffective, or (3) the assessed risk of material misstatement is low and other procedures address the risk.

The Smith Corporation uses prenumbered receiving reports that are released in numerical order from a locked box. For 2 days before the physical count all receiving reports are stamped "before inventory," and for 2 days after the physical count all receiving reports are stamped "after inventory." The receiving department continues to receive goods after the cutoff time while the physical count is in process. The least effective method for checking the accuracy of the cutoff is to A. Test trace receiving reports issued before the last receiving report to the physical items to see that they have been included in the physical count. B. List the number of the last receiving report for items included in the physical inventory count. C. Test trace receiving reports issued after the last receiving report to the physical items to see that they have not been included in the physical count. D. Observe that the receiving clerk is stamping the receiving reports properly.

Answer (D) is correct. The least effective method of checking the accuracy of the cutoff of inventory is to observe that the receiving clerk is stamping the receiving reports properly. The auditor is primarily concerned with the dates on the receiving reports to verify that goods received or shipped near year end are accounted for in the proper reporting period.

Which of the following procedures would an auditor least likely perform before the balance sheet date? A. Identification of related parties. B. Observation of merchandise inventory. C. Assessment of the risks of material misstatement. D. Confirmation of accounts payable.

Answer (D) is correct. The most important assertion about accounts payable is completeness, which is best tested at year end. For example, the auditor may examine subsequent cash payments to determine whether the related payables are not recorded. Although confirmation is not a required auditing procedure, it may be useful in detecting unrecorded payables if the auditor's sample includes vendors for which the risk of understatement is high, e.g., regular vendors with zero or low recorded balances.

The controller of Excello Manufacturing, Inc. wants to use ratio analysis to identify the possible existence of idle equipment or the possibility that equipment has been disposed of without having been written off. Which of the following ratios would best accomplish this objective? A. Repairs and maintenance cost to direct labor costs. B. Accumulated depreciation to the carrying amount of manufacturing equipment. C. Depreciation expense to the carrying amount of manufacturing equipment. D. Gross manufacturing equipment cost to units produced.

Answer (D) is correct. The ratio of gross manufacturing equipment cost to units produced increases if equipment is taken out of production but is not written off. As replacement equipment is installed, the amount of units produced remains constant, and the gross equipment cost increases. If equipment is taken out of service without being replaced, gross equipment cost remains constant, units produced decline, and the ratio again increases.

If management refuses to provide certain written representations that the auditor believes are essential, which of the following is appropriate? A. The auditor should express an adverse opinion because of management's refusal. B. The auditor can rely on oral evidence relating to the matter as a basis for an unmodified opinion. C. The client's refusal does not constitute a scope limitation that may lead to a modification of the opinion. D. The client's refusal may have an effect on the auditor's ability to rely on other representations of management.

Answer (D) is correct. The refusal constitutes a scope limitation that is often sufficient to preclude an unmodified opinion and may cause the auditor to disclaim an opinion or withdraw from the engagement. The auditor should also consider the effects of the refusal on his or her ability to rely on management's other representations (AU-C 580).

In gathering evidence in the performance of substantive procedures, the auditor most likely A. Considers the client's documentary evidence less reliable than evidence gathered orally by inquiry of management. B. Expresses an adverse opinion if (s)he has substantial doubt as to any assertion of material significance. C. Uses the test month approach. D. Relies on persuasive rather than conclusive evidence in the majority of cases.

Answer (D) is correct. To be appropriate, audit evidence should be relevant and reliable. Also, because of the inherent limitations of the audit, most audit evidence is persuasive rather than conclusive. However, although the cost of obtaining evidence and its usefulness should be rationally related, the matter of difficulty, time, or cost is not in itself a valid basis for (1) omitting a procedure when no alternative exists or (2) being satisfied with less than persuasive evidence.

Which of the following procedures would an auditor most likely perform in searching for unrecorded payables? A. Examine a sample of creditor balances to supporting invoices, receiving reports, and purchase orders. B. Review the responses of accounts receivable confirmations for indications of disputes with customers. C. Reconcile receiving reports with related cash payments made just prior to the year end. D. Compare cash payments made after the balance sheet date with the accounts payable trial balance.

Answer (D) is correct. Tracing subsequent payments to recorded payables is a primary procedure to match payments (checks issued) after year end with the related payables. Checks should be issued only for recorded payables. Any checks that cannot be matched are likely indications of unrecorded liabilities. Management may want to delay recording of liabilities to improve the current ratio. However, unrecorded accounts payable still must be paid, and financial statements that fail to report all liabilities at year end are misstated.

The auditor's inventory observation test counts are traced to the client's inventory listing to test for which of the following financial statement assertions? A. Presentation and disclosure. B. Rights and obligations. C. Valuation and allocation. D. Completeness.

Answer (D) is correct. Tracing the details of test counts to the final inventory schedule assures the auditor that items in the observed physical inventory are included in the inventory records.

An auditor will usually trace the details of the test counts made during the observation of the physical inventory taking to a final inventory schedule. This audit procedure is undertaken to provide evidence that items physically present and observed by the auditor at the time of the physical inventory count are A. Owned by the client. B. Not obsolete. C. Physically present at the time of the preparation of the final inventory schedule. D. Included in the final inventory schedule.

Answer (D) is correct. Tracing the details of test counts to the final inventory schedule assures the auditor that items in the observed physical inventory are included in the inventory records. The auditor should compare the inventory tag sequence numbers in the final inventory schedule to those in the records of his or her test counts made during the client's physical inventory.

Some subsequent events provide evidence of conditions not in existence at the balance sheet date. Under U.S. GAAP, some of these events are of such a nature that disclosure is required to keep the financial statements from being misleading. Adequate disclosure of these events may include A. Restatement of prior-period financial statements. B. Notes to the auditor's report. C. Adjustment of the financial statements. D. Pro forma financial statement presentation.

Answer (D) is correct. Under U.S. GAAP, subsequent events related to conditions that did not exist at the date of the balance sheet should not result in adjustments of (recognition in) the financial statements. These events are disclosed, if necessary, to keep the financial statements from being misleading. Occasionally, such an event may be so significant that disclosure can best be made by means of pro forma financial data. Such data make the event seem as if it had occurred on the date of the balance sheet. In some cases, U.S. GAAP suggest presentation of pro forma statements, usually a balance sheet only, in columnar form on the face of the historical statements. But firms usually incorporate the pro forma balance sheets in notes.

Which of the following tests of details most likely would help an auditor determine whether accounts payable have been misstated? A. Examining reported purchase returns that appear too low. B. Reviewing bank transfers recorded as cash received from customers. C. Searching for customer-returned goods that were not reported as returns. D. Examining vendor statements for amounts not reported as purchases.

Answer (D) is correct. Vendor statements should reflect currently recorded accounts payable. Examining statements at year end and matching line items to recorded payables will detect unrecorded payables

In performing tests of controls, the auditor will normally find that A. The level of inherent risk is directly proportional to the rate of error. B. The rate of error in the sample exceeds the rate of deviations. C. All unexamined items result in errors in the accounting records. D. The rate of deviations in the sample exceeds the rate of error in the accounting records.

Answer (D) is correct. When testing controls, the auditor is directly concerned with deviations from specific controls. Failure to comply with a control does not necessarily result in an error in the records. For example, the absence of an authorization signature does not necessarily mean that the transaction was improperly recorded. Accordingly, the rate of deviations from a control normally exceeds the error rate in the records.

When assessing the risks of material misstatement at a low level, an auditor is required to document the auditor's Understanding of the Entity's Control Environment Overall Responses to Assessed Risks

BOTH The understanding of the components of internal control, including the control environment, should be documented regardless of the degree of risk (AU-C 315). The overall responses to the assessed RMMs at the financial statement level also should be documented (AU-C 330).


Kaugnay na mga set ng pag-aaral

Lilley 36,37, Evolve questions for NUR140 Pharm - Ch.38, 39, 40 antibiotics, NCLEX Pharma Ch 41 AntiTubercular, Chapter 42 - Antifungal Drugs Elsevier Questions, Pharm chapter 43, Lilley Ch 44, Pharm. Ch. 45 (Antineoplastic Drugs Part 1), Chapter 46,...

View Set

Marketing test 3 chapter study guides 9,10,11,14

View Set

1 - ¿Cómo son? Write a sentence describing each subject using the appropriate adjective in parentheses. Follow the model. Modelo: (gordo, delgada) Lidia: Lidia es delgada. el novio de Olga: El novio de Olga es gordo.

View Set

Chapter 6 Microbial Metabolism: Fueling cell growth

View Set

Human Growth & Development Chapter 14

View Set

Chapter 12: Corporate Governance and Business Ethics

View Set